Question

32. Two charged particles are located on the x axis. The first is a charge 1Q at x 5 2a. The second is an unknown charge located at x 5 13a. The net electric field these charges produce at the origin has a magnitude of 2keQ /a2. Explain how many values are possible for the unknown charge and find the possible values
0 0
Add a comment Improve this question Transcribed image text
Know the answer?
Add Answer to:
32. Two charged particles are located on the x axis. The first is a charge 1Q...
Your Answer:

Post as a guest

Your Name:

What's your source?

Earn Coins

Coins can be redeemed for fabulous gifts.

Not the answer you're looking for? Ask your own homework help question. Our experts will answer your question WITHIN MINUTES for Free.
Similar Homework Help Questions
  • Two charged particles are located on the x axis. The first is a charge + of...

    Two charged particles are located on the x axis. The first is a charge + of 2k.Q/a. What are the two possible values of the unknown charge? negative value - posiive value + at x ฮ. The secund is an unknown charge l cated at x +30. The net electric field these charges produce at the origin has ส magnitude

  • r2 points 1 Previcus Two charged particles are located on the x aos. The first s...

    r2 points 1 Previcus Two charged particles are located on the x aos. The first s a char e + at x- a, The second is an unknown charge located at x·+3a The net electric neid these charges pr duce at the on in has a magnitude af 2keQ/a what are the two possible values of the unknown charge? negative value positive value fers from the correct answer by more than 10%. Double check your cálculations.x Q Your response differs...

  • Two charged particles, with charges q1=q and q2=4q, are located on the x axis separated by...

    Two charged particles, with charges q1=q and q2=4q, are located on the x axis separated by a distance of 2.00cm . A third charged particle, with charge q3=q, is placed on the x axis such that the magnitude of the force that charge 1 exerts on charge 3 is equal to the force that charge 2 exerts on charge 3. Find the position of charge 3 when q = 2.00 nC . Assuming charge 1 is located at the origin...

  • Two charged particles are placed along the x-axis. The first particle has a charge q1 =...

    Two charged particles are placed along the x-axis. The first particle has a charge q1 = +q and is at the origin. The second has a charge q2 = −2q and is at x = d = 4.60 cm. Determine the finite value of x (in cm) where the electric field is zero. Determine the smallest and largest finite values of x (in cm) where the electric potential is zero. (Note: Assume a reference level of potential V = 0...

  • A dipole is located at the origin, and is composed of charged particles with charge e...

    A dipole is located at the origin, and is composed of charged particles with charge e and -e, separated by a distance 2 10-10 m along the x axis. The positive x axis points to the right. The te charge is on the positive x-axis. Suggestion: draw a picture with the dipole and the observation location (where you want to find the field). What is the magnitude of the electric field due to this dipole at location 〈 0,2 *...

  • A dipole is located at the origin, and is composed of charged particles with charge e...

    A dipole is located at the origin, and is composed of charged particles with charge e and -e, separated by a distance 2 10-10 m along the x axis. The positive x axis points to the right. The te charge is on the positive x-axis. Suggestion: draw a picture with the dipole and the observation location (where you want to find the field). What is the magnitude of the electric field due to this dipole at location 〈 0,2 *...

  • Two charged particles are along the x axis: q1 is at -5.0 cm, and q2 is...

    Two charged particles are along the x axis: q1 is at -5.0 cm, and q2 is at 3.0 cm. The values of the charges are: q1= +4 microC q2= -2 microC Find the electric field (magnitude and direction) and the potential at the origin (at x = 0)..

  • Two 9.00 µC charged particles are located on the x axis. One is at x =...

    Two 9.00 µC charged particles are located on the x axis. One is at x = 1.00 m, and the other is at x = -1.00 m. (a) Determine the electric field on the y axis at y = 0.900 m. b) Calculate the electric force on a -3.00 µC charge placed on the y axis at y = 0.900 m.

  • Two 8.00 µC charged particles are located on the x axis. One is at x =...

    Two 8.00 µC charged particles are located on the x axis. One is at x = 1.00 m, and the other is at x = -1.00 m. (a) Determine the electric field on the y axis at y = 0.800 m. (b) Calculate the electric force on a -3.00 µC charge placed on the y axis at y = 0.800 m.

  • There are only two charged particles in a particular region. Particle 1 carries a charge of +q. Particle 2 carries a cha...

    There are only two charged particles in a particular region. Particle 1 carries a charge of +q. Particle 2 carries a charge of -2q. They are arranged on the x-axis as shown. Where is is possible to have the net field caused by these two charges equal to zero? Im thinking its at the origin maybe.... There are only two charged particles in a particular region. Particle 1 carries a charge of+ q. Particle 2 carries a charge of -2...

ADVERTISEMENT
Free Homework Help App
Download From Google Play
Scan Your Homework
to Get Instant Free Answers
Need Online Homework Help?
Ask a Question
Get Answers For Free
Most questions answered within 3 hours.
ADVERTISEMENT
ADVERTISEMENT
ADVERTISEMENT